Last visit was: 29 Apr 2024, 02:44 It is currently 29 Apr 2024, 02:44

Close
GMAT Club Daily Prep
Thank you for using the timer - this advanced tool can estimate your performance and suggest more practice questions. We have subscribed you to Daily Prep Questions via email.

Customized
for You

we will pick new questions that match your level based on your Timer History

Track
Your Progress

every week, we’ll send you an estimated GMAT score based on your performance

Practice
Pays

we will pick new questions that match your level based on your Timer History
Not interested in getting valuable practice questions and articles delivered to your email? No problem, unsubscribe here.
Close
Request Expert Reply
Confirm Cancel
SORT BY:
Date
Math Revolution GMAT Instructor
Joined: 16 Aug 2015
Posts: 10161
Own Kudos [?]: 16615 [0]
Given Kudos: 4
GMAT 1: 760 Q51 V42
GPA: 3.82
Send PM
Math Revolution GMAT Instructor
Joined: 16 Aug 2015
Posts: 10161
Own Kudos [?]: 16615 [0]
Given Kudos: 4
GMAT 1: 760 Q51 V42
GPA: 3.82
Send PM
Math Revolution GMAT Instructor
Joined: 16 Aug 2015
Posts: 10161
Own Kudos [?]: 16615 [0]
Given Kudos: 4
GMAT 1: 760 Q51 V42
GPA: 3.82
Send PM
Math Revolution GMAT Instructor
Joined: 16 Aug 2015
Posts: 10161
Own Kudos [?]: 16615 [0]
Given Kudos: 4
GMAT 1: 760 Q51 V42
GPA: 3.82
Send PM
Re: The Ultimate Q51 Guide [Expert Level] [#permalink]
Expert Reply
When n is divided by 4, the remainder is 3. If 3^n+2 is divided by 5, what is the remainder?

A. 1
B. 2
C. 3
D. 4
E. 0

==> You get n=4p+3, which becomes 3^n+2=3^{4p+3}+2=(3^4)^p(3^3)+2=(~1)^p(27)+2=(~1)(27)+2=(~7)+2=~9. Thus, the remainder when it is divided by 5 becomes 4.

The answer is D.
Answer: D
Math Revolution GMAT Instructor
Joined: 16 Aug 2015
Posts: 10161
Own Kudos [?]: 16615 [0]
Given Kudos: 4
GMAT 1: 760 Q51 V42
GPA: 3.82
Send PM
Re: The Ultimate Q51 Guide [Expert Level] [#permalink]
Expert Reply
What is the sum of all digits of 2^{13}5^7?

A. 3
B. 4
C. 6
D. 8
E. 10

==> You get 2^{13}5^7=2^6(2^75^7)=64(10^7)=640,000,000. The sum of the 0's at the back always becomes 0, so you get 6+4=10.

The answer is E.
Answer: E
Math Revolution GMAT Instructor
Joined: 16 Aug 2015
Posts: 10161
Own Kudos [?]: 16615 [0]
Given Kudos: 4
GMAT 1: 760 Q51 V42
GPA: 3.82
Send PM
Re: The Ultimate Q51 Guide [Expert Level] [#permalink]
Expert Reply
Is p/m>0?

1) p>m
2) pm>0.

==> If you modify the original condition and the question, from is p/m>0?, you multiply m^2 on both sides, you get (Squared number is always positive, so even if you multiply, the inequality sign doesn't change) p/m(m^2)>0(m^2)?, which becomes pm>0?.

The answer is B.
Answer: B
Math Revolution GMAT Instructor
Joined: 16 Aug 2015
Posts: 10161
Own Kudos [?]: 16615 [0]
Given Kudos: 4
GMAT 1: 760 Q51 V42
GPA: 3.82
Send PM
Re: The Ultimate Q51 Guide [Expert Level] [#permalink]
Expert Reply
If a, b, and c are positive integers, is (a+b)c divisible by 3?

1) 2-digit integer ab is divisible by 3.
2) When c is divided by 3, the remainder is 0.

==> If you modify the original condition and the question, in order to have (a+b)c to be divided by 3, a+b or c has to be divided by 3. However, if you look at con 2), c is divisible by 3, hence it is yes and sufficient. For con 1), ab is also divisible by 3, and thus a+b is also divisible by 3, hence it is yes and sufficient. Therefore, the answer is D. This type of question is a 5051-level question which applies CMT 4 (B: if you get A or B too easily, consider D).

Answer: D
Math Revolution GMAT Instructor
Joined: 16 Aug 2015
Posts: 10161
Own Kudos [?]: 16615 [0]
Given Kudos: 4
GMAT 1: 760 Q51 V42
GPA: 3.82
Send PM
Re: The Ultimate Q51 Guide [Expert Level] [#permalink]
Expert Reply
There are 5 apples in a bag. 4 apples are good but 1 apple is rotten. If you take out 2 apples from the bag, what is the probability that 1 apple selected is rotten?

A. 1/3
B. 1/4
C.2/5
D. 3/5
E.1/7

==> From probability=want/total, want=selecting one rotten apple and one good apple, and total=selecting 2 of 4 apples, so you get
probability=4C1*1C1/5C2=(4)(1)/(5*4/2!)=4/10=2/5.

The answer is C.
Answer: C
Math Revolution GMAT Instructor
Joined: 16 Aug 2015
Posts: 10161
Own Kudos [?]: 16615 [0]
Given Kudos: 4
GMAT 1: 760 Q51 V42
GPA: 3.82
Send PM
Re: The Ultimate Q51 Guide [Expert Level] [#permalink]
Expert Reply
There is a sequence of r, t, s, 1, 4, 3, 8, 15, 26. Each number of the sequence is the sum of the 3 previous numbers. r=?

A. -2
B. -1
C. 0
D. 1
E. 2

==> You get r+t+s=1, from t+s+1=4, you get t+s=3, and from r+t+s=r+3=1, you get r=1-3=-2.

The answer is A.
Answer: A
Math Revolution GMAT Instructor
Joined: 16 Aug 2015
Posts: 10161
Own Kudos [?]: 16615 [0]
Given Kudos: 4
GMAT 1: 760 Q51 V42
GPA: 3.82
Send PM
Re: The Ultimate Q51 Guide [Expert Level] [#permalink]
Expert Reply
When A works alone, it takes 14hrs, and when A works with B together, it takes 10hrs. How many hours does it take B to work alone?

A. 30hrs
B. 33hrs
C. 35hrs
D. 37hrs
E. 40hrs

==> For work rate questions, you solve “together and alone” reciprocally. It takes A 14hrs alone, and if you set the hours it took B to work alone as B hrs, from (1/14)+(1/B)=1/10 and 1/B=(1/10)-(1/14)=1/35, you get B=35.

The answer is C.
Answer: C
Math Revolution GMAT Instructor
Joined: 16 Aug 2015
Posts: 10161
Own Kudos [?]: 16615 [0]
Given Kudos: 4
GMAT 1: 760 Q51 V42
GPA: 3.82
Send PM
Re: The Ultimate Q51 Guide [Expert Level] [#permalink]
Expert Reply
If y≠0, is |x/y|=x/y?
1) |xy|=xy
2) |x/y|=|x|/|y|

==> If you modify the original condition and the question, in order to get |x/y|=x/y, you get x/y≥0?, and if you multiply y^2 on both sides, you get xy≥0?. Then, for con 1), to satisfy |xy|=xy, you get xy≥0, hence yes, it is sufficient. For con 2), if x=y=2, yes, but if x=2 and y=-1, no, hence it is not sufficient.

The answer is A.
Answer: A
Math Revolution GMAT Instructor
Joined: 16 Aug 2015
Posts: 10161
Own Kudos [?]: 16615 [0]
Given Kudos: 4
GMAT 1: 760 Q51 V42
GPA: 3.82
Send PM
Re: The Ultimate Q51 Guide [Expert Level] [#permalink]
Expert Reply
If x, y are positive integers, is xy=1?

1) x^2y^2=xy
2) 1/y=x.

==> In the original condition, there are 2 variables (x,y) and in order to match the number of variables to the number of equations, there must be 2 equations. Since there is 1 for con 1) and 1 for con 2), C is most likely to be the answer. By solving con 1) and con 2), for con 2), you get xy=1, hence it is unique and sufficient. For con 1), from (xy)^2-xy=0, xy(xy-1)=0, you get xy=0,1, but since x and y are positive, you get xy=1, and thus con 1) = con 2).

The answer is D.
Answer: D
Math Revolution GMAT Instructor
Joined: 16 Aug 2015
Posts: 10161
Own Kudos [?]: 16615 [0]
Given Kudos: 4
GMAT 1: 760 Q51 V42
GPA: 3.82
Send PM
Re: The Ultimate Q51 Guide [Expert Level] [#permalink]
Expert Reply
What is the remainder, when n(n+2) is divided by 24 for a positive integer x?

1) n is an even integer
2) n has remainder 0 or 1 when it is divided by 3.

=>Condition 1)
There are two kinds of even integers, which are n = 4k or n = 4k + 2 for some integer k. That means n could have 0 or 2 as a remainder when it is divided by n. If n = 4k, n(n+2) = 4k(4k+2) = 8k(2k+1) and n(n+1) is a multiple of 8. If n= 4k+2, n(n+2) = (4k+2)(4k+2+2) = 2(2k+1)*4(k+1) = 8(2k+1)(k+1) and n(n+1) is a multiple of 8. For both cases, n(n+1) is a multiple of 8. However, we can’t identify the remainder when it is divided by 3 from the condition 1).

Condition 2)
n = 3k or n = 3k +1. If n = 3k, n(n+2) = 3k(3k+2) is a multiple of 3. If n = 3k + 1, n(n+2) = (3k+1)(3k+3) = 3(3k+1)(k+1) is a multiple of 3. Thus, for both cases, n(n+1) is a multiple of 3. However, we can’t identify the remainder when it is divided by 8 from the condition 2).

Condition 1) & 2)
From the condition 1), n(n+1) is a multiple of 8. And n(n+1) is a multiple of 3 from the condition 2). Therefore, n(n+1) is a multiple of 24 from the both conditions 1) & 2) together.

Ans: C
Math Revolution GMAT Instructor
Joined: 16 Aug 2015
Posts: 10161
Own Kudos [?]: 16615 [0]
Given Kudos: 4
GMAT 1: 760 Q51 V42
GPA: 3.82
Send PM
Re: The Ultimate Q51 Guide [Expert Level] [#permalink]
Expert Reply
Is x=y?

1) x^4+y^4=2x^2y^2
2) x^4+y^4=0

==> In the original condition, there are 2 variables (x,y) and in order to match the number of variables to the number of equations, there must be 2 equations. Since there is 1 for con 1) and 1 for con 2), C is most likely to be the answer. By solving con 1) and con 2), for con 1), from x^4+y^4-2x^2y^2=0, (x^2-y^2)2=0, you get x^2=y^2, and from x=±y, yes and no coexists, hence it is not sufficient. For con 2), you only get x=y=0, hence yes, it is sufficient.

The answer is B.
Answer: B
Math Revolution GMAT Instructor
Joined: 16 Aug 2015
Posts: 10161
Own Kudos [?]: 16615 [0]
Given Kudos: 4
GMAT 1: 760 Q51 V42
GPA: 3.82
Send PM
Re: The Ultimate Q51 Guide [Expert Level] [#permalink]
Expert Reply
If 10^n< 0.000025 <10^{n+1}, what is the value of an integer n?

A. -5 B. -4 C. -3 D. 4 E. 5

=>0.00001 < 0.000025 < 0.0001
10^{-5} < 0.000025 < 10^{-4}

Ans: A
Math Revolution GMAT Instructor
Joined: 16 Aug 2015
Posts: 10161
Own Kudos [?]: 16615 [0]
Given Kudos: 4
GMAT 1: 760 Q51 V42
GPA: 3.82
Send PM
Re: The Ultimate Q51 Guide [Expert Level] [#permalink]
Expert Reply
Is the total profit from the sales of 3 products greater than $6?

1) The least profit from the sales of the 3 products is at least $2.1
2) The 2nd largest profit from the sales of the 3 products is at least $3.1.

=>Condition 1)
Since the minimum profit of 3 products is $2.1, the total profit of them is greater than or equal to $2.1 x 3 = $6.3, which is greater than $6.
Thus this is sufficient.

Condition 2)
Since the 3nd largest profit is at least $3.1, the sum of the first and the second products is at least $6.2, which is greater than $6.
Thus this is sufficient too.

Ans: D
Math Revolution GMAT Instructor
Joined: 16 Aug 2015
Posts: 10161
Own Kudos [?]: 16615 [0]
Given Kudos: 4
GMAT 1: 760 Q51 V42
GPA: 3.82
Send PM
Re: The Ultimate Q51 Guide [Expert Level] [#permalink]
Expert Reply
In a certain conference, if all the n attendees shake hands 105 times, what is the value of n?

A. 10 B. 12 C. 15 D. 16 E. 18

=>
nC2 = 105.
n(n-1)/(1*2) = 105
n(n-1) = 210
n(n-1) = 15*14

Thus n = 15.

Ans: C
Math Revolution GMAT Instructor
Joined: 16 Aug 2015
Posts: 10161
Own Kudos [?]: 16615 [1]
Given Kudos: 4
GMAT 1: 760 Q51 V42
GPA: 3.82
Send PM
Re: The Ultimate Q51 Guide [Expert Level] [#permalink]
1
Bookmarks
Expert Reply
If x-7=√x+√7 , x=?

A. 8+2√7 B. 8-2√7 C. 8+√7 D. 8-√7 E. 7+2√8

=>x-7=√x+√7
(√x+√7 )(√x-√7)=√x+√7
√x-√7=1
√x = 1 +√7
x = (1 +√7)2 = 8 +2√7

Ans: A
Math Revolution GMAT Instructor
Joined: 16 Aug 2015
Posts: 10161
Own Kudos [?]: 16615 [0]
Given Kudos: 4
GMAT 1: 760 Q51 V42
GPA: 3.82
Send PM
Re: The Ultimate Q51 Guide [Expert Level] [#permalink]
Expert Reply
If 11!/(11-r)!<1,000, what is the greatest possible value of r?

A. 1
B. 2
C. 3
D. 4
E. 5


=> 11! / (11-r)! = 11*(11-1)*…*(11-r+1)

r = 1 : 11 < 1,000
r = 2 : 11*10 = 110 < 1,000
r = 3 : 11*10*9 = 990 < 1,000
r = 4 : 11*10*9*8 = 7290 > 1,000

r = 3

Ans: C
Math Revolution GMAT Instructor
Joined: 16 Aug 2015
Posts: 10161
Own Kudos [?]: 16615 [0]
Given Kudos: 4
GMAT 1: 760 Q51 V42
GPA: 3.82
Send PM
Re: The Ultimate Q51 Guide [Expert Level] [#permalink]
Expert Reply
Is a positive integer x an odd number?

1) The smallest prime factor of x is 7.
2) The greatest prime factor of x is 7.

=>Condition 1)
2 cannot be a factor of x, since the smallest prime factor of x is 7.

Condition 2)
Consider n = 2 and n =3.


Ans: A
GMAT Club Bot
Re: The Ultimate Q51 Guide [Expert Level] [#permalink]
   1  ...  13   14   15   16   17   18   19  ...  64   

Powered by phpBB © phpBB Group | Emoji artwork provided by EmojiOne